maths help pls (1 Viewer)

scarvesss

howdy doo
Joined
May 28, 2011
Messages
162
Gender
Undisclosed
HSC
2012
the line y=mx is a tangent to the curve y=e^(3x). find m
 

scarvesss

howdy doo
Joined
May 28, 2011
Messages
162
Gender
Undisclosed
HSC
2012
the answer is apparently 3e. i would usually think to do the same thing but considering its a question 10 in trial hsc i think there is somewhere where we are going wrong
 

RealiseNothing

what is that?It is Cowpea
Joined
Jul 10, 2011
Messages
4,591
Location
Sydney
Gender
Male
HSC
2013
the answer is apparently 3e. i would usually think to do the same thing but considering its a question 10 in trial hsc i think there is somewhere where we are going wrong
3e can't be right, because that's a constant gradient, and hence that would mean is a linear function, which we know it's not.
 
Joined
May 18, 2012
Messages
193
Gender
Undisclosed
HSC
N/A
To be a tangent it must have a common point and the same gradient at that point

So

mx = e^(3x) ---- eqn 1 (Equating y values)

m= 3e^(3x) --- eqn 2 (Equating gradients)

Divide eqn 2 by eqn 1 (note we can do this as e^(3x) is never zero)

m/(mx) = [3e^(3x) /e^(3x) ]

1/x = 3

x=1/3


Now sub back into eqn 2

m= 3e^( 3*(1/3))

m=3e
 
Last edited:

nightweaver066

Well-Known Member
Joined
Jul 7, 2010
Messages
1,585
Gender
Male
HSC
2012
3e can't be right, because that's a constant gradient, and hence that would mean is a linear function, which we know it's not.
It doesn't mean that e^3x has to be a linear function.. The line is just a tangent and 3e is correct.
 

scarvesss

howdy doo
Joined
May 28, 2011
Messages
162
Gender
Undisclosed
HSC
2012
holy shit fortune_cookie* you killed it. never would have thought to do that. and im glad others didnt as well
 
Last edited:

Drongoski

Well-Known Member
Joined
Feb 22, 2009
Messages
4,249
Gender
Male
HSC
N/A
m = 3e is correct.

If you think of it: y = e3x is y = (e3)x is just anothe exponential function roughly y = 20.0855x

Its graph looks like the typical one for, say, y = 2x crossing the y-axis at 1. y = mx is just a straight line thru the origin. Only one such straight line (i.e. for only one value of m) can touch the graph of y = e^3x and that occurs at where x = 1/3 for m = 3e as ably worked out by Mr fortune-cookie.

Edit

Some of you more able than I can show the graphs of the exponential function and the tangent line y = mx.
 
Last edited:

Users Who Are Viewing This Thread (Users: 0, Guests: 1)

Top